0% found this document useful (0 votes)
13 views76 pages

AP Physics 2 2015 2022 Frq Ans

The document contains the 2015 and 2016 scoring guidelines for the AP Physics 2 exam, detailing the point distribution for various questions and parts. It outlines the expectations for student responses, including calculations, explanations, and reasoning related to physics concepts such as light behavior, electric fields, and gas laws. Each question is broken down into parts with specific criteria for earning points, emphasizing the importance of clear reasoning and accurate calculations.

Uploaded by

hanmin.ryu
Copyright
© © All Rights Reserved
We take content rights seriously. If you suspect this is your content, claim it here.
Available Formats
Download as PDF, TXT or read online on Scribd
0% found this document useful (0 votes)
13 views76 pages

AP Physics 2 2015 2022 Frq Ans

The document contains the 2015 and 2016 scoring guidelines for the AP Physics 2 exam, detailing the point distribution for various questions and parts. It outlines the expectations for student responses, including calculations, explanations, and reasoning related to physics concepts such as light behavior, electric fields, and gas laws. Each question is broken down into parts with specific criteria for earning points, emphasizing the importance of clear reasoning and accurate calculations.

Uploaded by

hanmin.ryu
Copyright
© © All Rights Reserved
We take content rights seriously. If you suspect this is your content, claim it here.
Available Formats
Download as PDF, TXT or read online on Scribd
You are on page 1/ 76

AP Physics 2

2015 Scoring Guidelines

© 2015 The College Board. College Board, Advanced Placement Program, AP, AP Central, and the acorn logo
are registered trademarks of the College Board.
Visit the College Board on the Web: www.collegeboard.org.
AP Central is the official online home for the AP Program: apcentral.collegeboard.org.
AP® PHYSICS 2
2015 SCORING GUIDELINES

Question 1
10 points total Distribution
of points
(a) 5 points

For explaining that the light that results in spot X reflects at the glass-liquid 1 point
interface
For explaining that the light that results in spot Y is refracted both as it leaves the 1 point
glass and reenters the glass (direction of refraction need not be correct)
For explaining that the light that results in spot Y is reflected at the liquid-air 1 point
interface
For explaining that Y is farther from P than X in terms of the geometry of the path 1 point
For explaining that at each interface the brightness is affected by reflection, and 1 point
by transmission and/or refraction as appropriate
Students may reference a diagram to support the reasoning, but it cannot supplant
the written reasoning.

(b)

i) 2 points

For drawing a reasonably accurate path for the rays that form spot X 1 point
For drawing a reasonably accurate path for the rays that form spot Y, including 1 point
correct directions of refraction, with no rays in the air

ii) 1 point

For indicating that Y becomes brighter in terms of the distribution of energy with 1 point
and without total internal reflection

© 2015 The College Board.


Visit the College Board on the Web: www.collegeboard.org.
AP® PHYSICS 2
2015 SCORING GUIDELINES

Question 1 (continued)
Distribution
of points
(c)

i) 1 point

For drawing rays to show reflection at the glass-liquid interface 1 point


One point will be deducted for drawing any rays above the glass-liquid interface.

ii) 1 point

For indicating that Y disappears and indicating that total internal reflection takes 1 point
place at the glass-liquid interface

© 2015 The College Board.


Visit the College Board on the Web: www.collegeboard.org.
AP® PHYSICS 2
2015 SCORING GUIDELINES

Question 2
12 points total Distribution
of points
(a)
i) 3 points

For indicating that Req of the entire circuit or the combination of bulbs 2 and 3 1 point
decreases
For indicating a change in I tot or the potential difference across bulb 1 consistent 1 point
with Ohm’s law and the change in Req stated in the response
For indicating a change in brightness consistent with the current or potential 1 point
difference change stated in the response

ii) 3 points

For indicating that P1 =


4(
1 2
e R ) 1 point

For indicating that the new equivalent resistance of the circuit is Req,new = (3 2) R 1 point
Note: Credit is earned if calculation is done in part (i) and used here.
For manipulating equations to show that the power expended by bulb 1 is 1 point
16
Pnew = P
9 1

iii) 1 point

For using or referring to the expression from part (a)(ii) to support the claim made 1 point
in (a)(i) regarding the brightness of bulb 1: e.g., 16 9 > 1 , and indicating an
understanding that brightness is related to power consumption

(b)

i) 1 point

For explaining that the brightness of bulb 2 decreases after the switch is closed 1 point
because it expends less power (or the current through bulb 2 decreases, or the
potential difference across bulb 2 decreases)

ii) 1 point

For a calculation that supports the reasoning in part i 1 point

© 2015 The College Board.


Visit the College Board on the Web: www.collegeboard.org.
AP® PHYSICS 2
2015 SCORING GUIDELINES

Question 2 (continued)
Distribution
of points
(c) 3 points

For indicating in either part (c)i or part (c)ii that brightness is dependent on 1 point
potential difference across the bulb OR on current through the bulb

i)
For a reasonable explanation for why bulb 1 is brighter than bulb 2 1 point
Example: Immediately after the switch is closed, the potential difference across the
capacitor will be zero (like a short in the circuit), so the current through bulb 2
would be zero, which is less than the current through bulb 1.
Note: No points will be awarded for indicating that bulb1 is brighter than bulb 2
with no justification.

ii)
For a reasonable explanation for why bulb 1 is the same brightness as bulb 2 1 point
Example: The current through bulb 2 increases as the potential difference across
the capacitor increases (becomes like an open circuit), so a long time after the
switch is closed, the current through bulb 2 will be equal to the full current
through bulb 1.
Note: No points will be awarded for indicating that bulb 1 is the same brightness
as bulb 2 without a justification.

© 2015 The College Board.


Visit the College Board on the Web: www.collegeboard.org.
AP® PHYSICS 2
2015 SCORING GUIDELINES

Question 3
12 points total Distribution
of points
(a) 2 points

For indicating that the ideal gas law ( PV = nRT or PV = NkT ) gives the relevant 1 point
relationship between pressure and temperature of a gas and attempting to use
it to support some reasoning
For indicating that the volume and number of moles (or particles) of gas are held 1 point
constant
Note: The student will not be penalized for not specifying that pressure and
temperature are only directly proportional when the temperature is measured
in Kelvin.
Alternate Solution Alternate Points
For indicating that the density of a sample of gas increases as its temperature 1 point
decreases (if the pressure and number of moles or molecules of gas are held
constant), and a sample of denser gas will sink below samples of gas that are
less dense
For indicating that the gas near the North Pole is not a closed system and its 1 point
pressure will increase as additional sinking gas molecules are added to it

(b) 4 points

For selecting one of the cylinders and indicating or implying that volume is held 1 point
constant
For selecting all the equipment described in the procedure and no extraneous 1 point
equipment
For describing a method of measuring the temperature of the enclosed gas 1 point
For describing a method for measuring the pressure at more than just two 1 point
temperatures
Example: Insert the thermometer and pressure sensor in the gasket to measure the
gas temperature and pressure. Place the cylinder in the bath with hot (cold)
water. Take measurements periodically as the bath water cools (heats) over
time.

© 2015 The College Board.


Visit the College Board on the Web: www.collegeboard.org.
AP® PHYSICS 2
2015 SCORING GUIDELINES

Question 3 (continued)
Distribution
of points
(c) 2 points

For selecting a set of trials in which volume is held constant and explaining that 1 point
the volume must be held constant to test the relationship between pressure
and temperature
For selecting trials in which volume is 5.0 cm3 , and explaining that there are the 1 point
most trials for this volume, and the most trials will result in the most reliable
test
Alternate Solution Alternate points
For selecting the full set of trials and explaining that the effect of changing volume 1 point
on the relationship between pressure and temperature can be taken into
account by multiplying pressure by volume (or plotting P T as a function of
1 V , etc.)
For explaining that selecting the most trials will result in the most reliable test OR 1 point
that selecting the widest range of pressure values will result in the most
precise determination of the proportionality constant relating pressure and
temperature

(d) 3 points

For plotting P as a function of T (or T as a function of 1 P , etc.) OR plotting PV as a 1 point


function of T (or P as a function of V T , etc.) for each trial selected in part (c)
For appropriate axis labels with units and appropriate scales 1 point
For drawing an appropriate best-fit line or curve 1 point
Example:

© 2015 The College Board.


Visit the College Board on the Web: www.collegeboard.org.
AP® PHYSICS 2
2015 SCORING GUIDELINES

Question 3 (continued)
Distribution
of points

(e) 1 point

For correctly describing the relationship depicted in part (d) 1 point


Examples:
The relationship between P and T is linear.
The relationship between P and V T is hyperbolic.

© 2015 The College Board.


Visit the College Board on the Web: www.collegeboard.org.
AP® PHYSICS 2
2015 SCORING GUIDELINES

Question 4
10 points total Distribution
of points
(a) 2 points

The top plate is negative.


For relating the direction of force or acceleration to the direction of the field 1 point
For relating the direction of the electric field to the sign of the charge on the top 1 point
plate
No points are awarded for identifying that the top plate is negative with no
attempt to explain why.

(b) 4 points

For using an appropriate kinematic relation to determine the acceleration of the 1 point
electron while it is between the plates
a = ( v f - vi ) t
For using Newton’s second law to determine an expression for the magnitude of 1 point
the force needed to give the electron the calculated acceleration
F = ma = m ( v f - vi ) t
For setting eE equal to the force calculated from Newton’s second law 1 point
For correctly manipulating equations to solve for the magnitude of the electric field 1 point
and arriving at a correct numerical answer with units
E = m ( v f - vi ) et

E =
(9.11 ¥ 10-31 kg)(8.02 ¥ 106 m s - 5.40 ¥ 106 ms ) = 10,000 N C
(1.6 ¥ 10-19 C)(1.49 ¥ 10-9 s)
Alternate Solution Alternate Points
For applying conservation of energy for the time the electron is between the plates 1 point
DK = DU
For using the correct relationship between potential energy and potential 1 point
difference
DU = e DV
1
( )
m v 2 - vi2 = e DV
2 e f
For using the relation between potential difference, electric field, and plate 1 point
separation
DV = Ed
1
( )
m v 2 - vi2 = e Ed
2 e f
For correctly manipulating equations to solve for the magnitude of the electric field 1 point
and arriving at a correct numerical answer with units
( )
E = me v 2f - vi2 2 e d

(9.11 ¥ 10-31 ) ((8.02 ¥ 106 m s) - (5.40 ¥ 106 m s) )


2 2

E = = 10,000 N C
2 (1.6 ¥ 10-19 C) (0.010 m)

© 2015 The College Board.


Visit the College Board on the Web: www.collegeboard.org.
AP® PHYSICS 2
2015 SCORING GUIDELINES

Question 4 (continued)
Distribution
of points
(c) 1 point

E = Q e0 A
Q = e0 AE
For correct substitution of values into the equation to calculate the magnitude of 1 point
charge on each parallel plate
( )( )
Q = 8.85 ¥ 10 -12 C 2 N  m 2 0.25 m 2 (10,000 N C )
Q = 2.2 ¥ 10-8 C

(d)
i. 2 points

For drawing a reasonably circular path from the point where the electrons leave 1 point
the bottom plate to point X
Note: There is no penalty for starting the path at the tip of the arrow.
For explaining that the field is always perpendicular to the velocity, so the force is 1 point
also always perpendicular to the velocity which creates a curved (circular) path

ii. 1 point

In order for the electron to reach point X, the magnetic field must exert a
centripetal force on the electron toward the top right corner of the dashed box.
For using the right hand rule and reasoning that the force on a negatively charged 1 point
object is in the opposite direction from the force exerted on a positively
charged object (or using the “left hand rule”) to conclude that the direction of
the magnetic field is directed out of the page
Notes:
No points are awarded for identifying that the direction of the magnetic field is
out of the page without explaining why.
Credit can be earned for a correct analysis at any individual point on the path.

© 2015 The College Board.


Visit the College Board on the Web: www.collegeboard.org.
AP Physics 2
®

2016 Scoring Guidelines

© 2016 The College Board. College Board, Advanced Placement Program, AP, AP Central, and the acorn logo
are registered trademarks of the College Board.
Visit the College Board on the Web: www.collegeboard.org.
AP Central is the official online home for the AP Program: apcentral.collegeboard.org.
AP® PHYSICS 2
2016 SCORING GUIDELINES

Question 1
10 points total Distribution
of points
(a)
i. 2 points

For showing the calculation of the force on the piston and a correct answer with 1 point
units
( )(
F = PA = 1.0 ¥ 10 5 Pa 5 ¥ 10 -3 m 2 = 500 N )
For explaining the force in terms of gas atom collisions — some change in the 1 point
atoms’ momentum or velocity must be identified to justify a force between
atoms and piston
Example: The collisions of the gas atoms with the container walls cause a change
in the momentum of the gas atoms, which means forces are exerted between
the atoms and the piston. Each gas molecule colliding with a wall experiences
a force from the wall that changes the molecule’s velocity or momentum.

ii. 2 points

For showing the calculation of the temperature and a correct answer with units 1 point
PV = nRT
(
T = PV nR = 1.0 ¥ 10 5 Pa 0.10 m 3 )( ) (2)(8.31 J molK ) = 602 K
For indicating that temperature characterizes the average speed or average kinetic 1 point
energy or RMS velocity of the molecules

(b)
i. 2 points

For identifying that the temperature increases due to increasing volume and 1 point
constant pressure
For relating temperature change with internal energy change 1 point
Example: Because the volume increases at a constant pressure, the temperature
goes up because PV = nRT . Increasing temperature means increasing
average kinetic energy or total internal energy.

ii. 3 points

For calculating the work done in process ABC (i.e., the area under the line) 1 point
( )(
WAB = - 1.0 ¥ 10 Pa 0.10 m - 0.04 m
5 3 3
) = -6000 J and WBC = 0
For calculating TA and TC (or DT between the states) and using them to 1 point
determine internal energy change
(
TA = PAVA nR = 1.0 ¥ 10 5 Pa 0.04 m 3)( ) (2 mol)(8.31 J molK ) = 241 K
TC = PC VC nR = ( 0.5 ¥ 10 5 Pa )( 0.10 m 3 ) (2 mol)(8.31 J molK ) = 301 K
DU = DK per molecule nN 0 = (3 2) kB DT nN 0

( ) (
DU = (3 2 ) 1.38 ¥ 10 -23 J K (301 K - 241 K )(2 mol) 6.02 ¥ 10 23 = 1500 J )

© 2016 The College Board.


Visit the College Board on the Web: www.collegeboard.org.
AP® PHYSICS 2
2016 SCORING GUIDELINES

Question 1 (continued)
Distribution
of points
(b)
ii. (continued)

Alternately, U can be calculated directly from the given data


DU = (3 2 ) nRDT = (3 2 ) ( PC VC - PAVA )
(( )( ) ( )(
= (3 2 ) 0.5 ¥ 10 5 Pa 0.10 m 3 - 1.0 ¥ 10 5 Pa 0.04 m 3 )) = 1500 J
For substituting U and W (whether correct or incorrect) into some form of the first 1 point
law of thermodynamics to find Q and for including units in a numerical answer
Q = DU - W = 1500 J - ( -6000 J )
Q = 7500 J

(c) 1 point

For recognizing that the change in kinetic energy for process CA has the same 1 point
numerical value as U from (b)ii but with the opposite sign OR for calculating
DK using the correct temperature change or K total = (3 2 ) nR T as shown
below
K total = (3 2 ) kB T nN 0 or K total = (3 2 ) nR T
( ) (
K total = (3 2 ) 1.38 ¥ 10 -23 J K (241 K - 301 K )( 2 mol) 6.02 ¥ 10 23 mol -1 )
K total = -1500 J

© 2016 The College Board.


Visit the College Board on the Web: www.collegeboard.org.
AP® PHYSICS 2
2016 SCORING GUIDELINES

Question 2
12 points total Distribution
of points
(a) 3 points

For graphing angles or functions of angles on the axes 1 point


For plotting sines of angles on the axes and indicating or implying that the index of 1 point
refraction of air is 1
For indicating a method to determine the index of refraction of the glass that is 1 point
consistent with the graph described
Example: If 1 refers to air and 2 to the glass, use n1 sin q1 = n2 sin q2 and graph
sin q1 as a function of sin q2 . Because n1 = 1 , the slope of the line is n2 .

(b) 4 points

For indicating that the light from the lamp needs to be a beam when it enters the 1 point
glass (either referring to a beam, explicitly describing how to create a beam, or
showing a beam in a diagram)
For describing some method of determining angles with a protractor (or equivalent 1 point
tool) in both media at an appropriate boundary
For using angles with respect to the normal (can measure any angles as long as 1 point
reference is made to converting them to the correct ones)
For repeating the measurement at three or more different incidence angles to 1 point
obtain sufficient data

(c) 2 points

For indicating that when light travels across the boundary from air to glass, the ray 1 point
bends toward the normal
For indicating that the speed of light is slower in glass than in air (or an answer 1 point
consistent with response for bending of the ray)

© 2016 The College Board.


Visit the College Board on the Web: www.collegeboard.org.
AP® PHYSICS 2
2016 SCORING GUIDELINES

Question 2 (continued)
Distribution
of points
(d) 3 points

For an appropriate energy-level diagram showing absorption from the lowest level 1 point
and emission
For indicating that a hydrogen atom can be excited from the ground state to a 1 point
higher energy state by absorbing energy
For indicating that transitions to lower energy levels cause emission of photons 1 point
Example:

Photon
emission
Energy
absorption

Ground state

Atoms in the ground state absorb energy from the electricity delivered to the
lamp. The atoms enter an excited state. Then the atoms emit photons as
they drop to a lower energy state.

© 2016 The College Board.


Visit the College Board on the Web: www.collegeboard.org.
AP® PHYSICS 2
2016 SCORING GUIDELINES

Question 3
12 points total Distribution
of points
(a) 1 point

For correctly labeling the magnitude of lines ( A = 60 V and B = 80 V , units not 1 point
required) and for having the correct signs on each line (either explicit positive
signs or no negative signs)

(b)

i. 2 points

For indicating that Y has more charge than X with a correct explanation, such as 1 point
the nearest potential of the same value is farther from Y
For indicating X and Y must be the same sign with a correct explanation 1 point
Examples:
 Field vectors are perpendicular to equipotentials, and the pattern of field
vectors indicates that both must have the same sign.
 There is no zero potential line between the spheres, so the potentials from
the two charges do not cancel anywhere.

ii. 1 point

For a correct explanation that addresses the charge to mass ratio and distance 1 point
relationship
Example: Both spheres would gravitationally attract a third sphere just like two
charges of the same sign would attract a third charge of the opposite sign
because both forces are dependent on the distance and also dependent on the
product of the charges or masses.

(c) 2 points

For a similarity that does not generally apply to all forces 1 point
Example: The forces have the same dependence on the distance and are both
functions of 1 r 2 .
For a difference that does not generally apply to all forces 1 point
Examples:
The proton is the same sign as the spheres, so it is electrostatically repelled, but
gravity is always attractive. Therefore, the directions of the forces are
different.
The forces are different in magnitude.

© 2016 The College Board.


Visit the College Board on the Web: www.collegeboard.org.
AP® PHYSICS 2
2016 SCORING GUIDELINES

Question 3 (continued)
Distribution
of points
(d)

i. 2 points

The proton is repelled by both spheres, so it will move toward point A.


For some attempt to apply DU E = qDV using the given variables 1 point
For using the correct variables and getting DU = q (VA - VB ) (no credit for using 1 point
numeric values of electric potential)

ii. 2 points

For using a correct expression for work 1 point


W = Fd or - DU E = qDV
For a correct expression for F or DV in terms of Eavg 1 point
Favg = qEavg or DV = Eavg d
W = qEavg d (full credit is awarded for just writing the correct expression)

iii. 2 points

For indicating that student 1 is correct and a correct discussion of why 1 point
Example: Student 1 is correct. Because the system contains all the objects, the
energy is transferred from one form to another within the system due to the
law of conservation of energy.
For indicating that student 2 is correct and a correct discussion of why 1 point
Example: Student 2 is correct. Because the system is just the proton, there must be
a force external to the system due to the electric field of the two spheres. That
force does work on the proton which changes the kinetic energy of the proton.

© 2016 The College Board.


Visit the College Board on the Web: www.collegeboard.org.
AP® PHYSICS 2
2016 SCORING GUIDELINES

Question 4
10 points total Distribution
of points
(a)

i. 1 point

For indicating that I = e R and VC = 0 1 point


Because there is no charge on the capacitor, there is no potential difference across
it. Therefore, entire battery potential is across the resistor, so the current is
that potential divided by the resistance.

ii. 1 point

For indicating that I = 0 and VC = e 1 point


Once the capacitor is fully charged, it allows no current to pass. Because all the
components are in series, there is no current at all in the circuit. With no
current, there is no potential difference across the resistor, so the entire battery
potential is across the capacitor.

(b)

i. 2 points

For a calculation that indicates one of the following: 1 point


 The potential difference across each capacitor in the new circuit is half
that across the single capacitor in the original circuit
 The equivalent capacitance of the new circuit is one-half the capacitance
of the original circuit
U1 = (1 2) C e 2
U 2 = 2 ÈÎ(1 2) C ( e 2)2 ˘˚ or (1 2)(C 2) e 2 , which both equal C e 2 4
For correctly calculating the ratio 1 point
( )(
U1 U 2 = C e 2 2 C e 2 4 = 2 )
ii. 1 point

For any combination of area and spacing that is consistent with the student’s 1 point
answer for the ratio in part (b)(i), with a proper principle or model as support
Example: U = (1 2) CV 2 . The potential difference across each of the single
capacitors is the same. For the energy stored in the single new capacitor to be
half that of the original single capacitor, the new capacitor must have half the
capacitance. C = e0 A d , so half the plate area with the same distance
between the plates will accomplish this.

© 2016 The College Board.


Visit the College Board on the Web: www.collegeboard.org.
AP® PHYSICS 2
2016 SCORING GUIDELINES

Question 4 (continued)
Distribution
of points
(c) 5 points

Arrangement 1 Arrangement 2

For two correct circuit diagrams, each matched with the correct situation — 1 point
arrangement 1 has lightbulb and capacitor in parallel, and arrangement 2 has
them in series

For indicating that the lightbulb is brightest when the current through it is maximum 1 point
and that the capacitor eventually stops current from flowing in its branch when
the potential difference across its plates is equal in magnitude to the emf of the
battery (or something similar)

Response using current Response using potential difference


For indicating that in the series circuit For indicating that in the series circuit 1 point
(where the same current flows (where the potential is shared) the
through both components) the most resistor has its maximum potential
current flows right after the switch is difference right after the switch is
closed and decreases as the capacitor closed, because the capacitor starts
charges out uncharged (no potential
difference) and then charges until it
has the same potential as the battery

For indicating that in the parallel circuit For indicating that in a parallel circuit 1 point
(where the current is shared between (where both components have the
the components) the most current same potential difference) the bulb
flows through the lightbulb a long starts out with the same zero
time after the switch is closed, potential difference as the capacitor
because the full current initially goes and ends up with the total battery
through the capacitor branch because potential
it acts like a wire (very low potential
difference), then ends up all through
the lightbulb once the fully charged
capacitor acts like an open circuit
(same potential difference as battery)

For a response that has sufficient paragraph structure, as described in the published 1 point
requirements for the paragraph-length response

© 2016 The College Board.


Visit the College Board on the Web: www.collegeboard.org.
2017

AP Physics 2:
Algebra-Based
Scoring Guidelines

© 2017 The College Board. College Board, Advanced Placement Program, AP, AP Central, and the acorn logo
are registered trademarks of the College Board. Visit the College Board on the Web: www.collegeboard.org.
AP Central is the official online home for the AP Program: apcentral.collegeboard.org
AP® PHYSICS
2017 SCORING GUIDELINES

General Notes About 2017 AP Physics Scoring Guidelines

1. The solutions contain the most common method of solving the free-response questions and the
allocation of points for this solution. Some also contain a common alternate solution. Other methods of
solution also receive appropriate credit for correct work.

2. The requirements that have been established for the paragraph length response in Physics 1 and
Physics 2 can be found on AP Central at
https://secure-media.collegeboard.org/digitalServices/pdf/ap/paragraph-length-response.pdf.

3. Generally, double penalty for errors is avoided. For example, if an incorrect answer to part (a) is
correctly substituted into an otherwise correct solution to part (b), full credit will usually be awarded.
One exception to this may be cases when the numerical answer to a later part should be easily
recognized as wrong, e.g., a speed faster than the speed of light in vacuum.

4. Implicit statements of concepts normally receive credit. For example, if use of the equation expressing a
particular concept is worth one point, and a student’s solution embeds the application of that equation to
the problem in other work, the point is still awarded. However, when students are asked to derive an
expression it is normally expected that they will begin by writing one or more fundamental equations,
such as those given on the exam equation sheet. For a description of the use of such terms as “derive”
and “calculate” on the exams, and what is expected for each, see “The Free-Response SectionsStudent
Presentation” in the AP Physics; Physics C: Mechanics, Physics C: Electricity and Magnetism Course
Description or “Terms Defined” in the AP Physics 1: Algebra-Based and AP Physics 2: Algebra-Based
Course and Exam Description.

5. The scoring guidelines typically show numerical results using the value g = 9.8 m s 2 , but use of
10 m s 2 is of course also acceptable. Solutions usually show numerical answers using both values when
they are significantly different.

6. Strict rules regarding significant digits are usually not applied to numerical answers. However, in some
cases answers containing too many digits may be penalized. In general, two to four significant digits are
acceptable. Numerical answers that differ from the published answer due to differences in rounding
throughout the question typically receive full credit. Exceptions to these guidelines usually occur when
rounding makes a difference in obtaining a reasonable answer. For example, suppose a solution requires
subtracting two numbers that should have five significant figures and that differ starting with the fourth
digit (e.g., 20.295 and 20.278). Rounding to three digits will lose the accuracy required to determine the
difference in the numbers, and some credit may be lost.

© 2017 The College Board.


Visit the College Board on the Web: www.collegeboard.org.
AP® PHYSICS 2
2017 SCORING GUIDELINES

Question 1
10 points total Distribution
of points
(a)
i. 1 point

For indicating that student Y is correct in stating that the water moves faster at point B, 1 point
and not indicating any other aspect

ii. 2 points

Student Y’s statement that PB is greater than PA is not correct.


For a correct indication of how height affects pressure using the Bernoulli equation 1 point
(i.e., conservation of energy principles)
For correct indication of how the speed affects pressure using the Bernoulli equation 1 point
(i.e., conservation of energy principles)
Example: The pressure at point B is not greater. Because the water at B is moving faster
and is higher than at point A, the kinetic energy and the gravitational potential
energy terms in Bernoulli’s equation are both greater. Because the sum of pressure
and these energy terms is a constant, the pressure must be less.

iii. 1 point

For indicating one of the following: 1 point


 Student Z is correct in stating that the potential energy of the water-Earth
system has increased.
 Student Z is correct in stating that conservation of energy applies.
 Stating that nothing is correct or giving no response, with a justification
in (iv).

iv. 1 point

For indicating that student Z is incorrect in stating that the speed is less at point B, not 1 point
indicating any other aspect, and using continuity or the Bernoulli equation (i.e.,
conservation of energy principles) to show that it is greater
OR
if third bullet for (iii) applies, indicating that work is done on the water due to the
pressure difference, so the energy is not constant

© 2017 The College Board.


Visit the College Board on the Web: www.collegeboard.org.
AP® PHYSICS 2
2017 SCORING GUIDELINES

Question 1 (continued)
Distribution
of points
(b)
i. 2 points

For a correct application of the continuity equation including substitutions 1 point


A A v A = AB v B
vB = AAv A AB = rA2 v A rB2 = ( 2.5 cm )2 (0.5 m s) (1.5 cm )2
For a correct answer with units 1 point
v B = 1.4 m s

ii. 1 point

For an application of Bernoulli’s equation to this situation and substitutions consistent 1 point
with (b)(i)
1 1
PA + rgy A + rv 2A = PB + rgyB + rvB2
2 2
1
(
PB = PA + rg ( y A - yB ) + r v 2A - vB2
2 )
1
 
PB  2  10  1000 10  5   1000  0.52  1.42  2  105  50000  855
5
2
PB = 1.5 ¥ 105 Pa

(c)
i. 1 point

For substituting correctly in an appropriate equation for determining the pressure 1 point
( )(
P = P0 + r gh A = 1 ¥ 105 Pa + 1000 kg m 3 10 m s 2 (6 m ) )
P = 1.6 ¥ 105 Pa

ii. 1 point

For indicating that the buoyant force is toward the top of the page and gravity is toward 1 point
the bottom of the page, with the buoyant force longer
Student can draw lots of pressure forces around the dot instead of one buoyant force, as
long as there is no buoyant force labeled and they add up to a net buoyant force that
is longer than the gravitational force.

© 2017 The College Board.


Visit the College Board on the Web: www.collegeboard.org.
AP® PHYSICS 2
2017 SCORING GUIDELINES

Question 2
12 points total Distribution
of points
(a)
i. 5 points

For drawing a circuit with the battery, rod, and ammeter in series (rods can be drawn to 1 point
look like rods, or schematically as resistors)
For drawing the voltmeter parallel to the rod, or indicating that the setting on the power 1 point
supply will be used
For measuring potential difference and current for a rod 1 point
For measuring the length and diameter of a rod 1 point
For including multiple trials with appropriate controls 1 point
Examples: 1) Use one rod and apply different potential differences
2) Use different rods

ii. 2 points

For graphing appropriate quantities whose slope can be used to calculate resistance 1 point
directly or indirectly
For correctly stating how the slope relates to the resistivity 1 point

© 2017 The College Board.


Visit the College Board on the Web: www.collegeboard.org.
AP® PHYSICS 2
2017 SCORING GUIDELINES

Question 2 (continued)
Distribution
of points
(b) 3 points

5
Potential Difference (V)

0
0 0.01 0.02 0.03 0.04 0.05 0.06 0.07 0.08
Current (A)

For plotting the data on the graph with potential difference on one axis and current on 1 point
the other, labeled with units, using a reasonable scale
For a clearly shown calculation of slope from a reasonable best-fit line 1 point
For a correct answer with units 1 point
Acceptable range is 70 - 79 W .
For a graph of I as a function of V, the slope should end up near 0.013 1 and the
resistance is the inverse
For a graph of V as a function of I, the slope should end up near 74  and equals the
resistance
For the example shown above
slope =
(5.8 - 2.4) V = 73.9 W
( 0.078 - 0.032) A

© 2017 The College Board.


Visit the College Board on the Web: www.collegeboard.org.
AP® PHYSICS 2
2017 SCORING GUIDELINES

Question 2 (continued)
Distribution
of points

(c)
i. 1 point

For indicating that the internal resistance of the power supply will not affect the data 1 point
acquired, with correct reasoning
Example: Because potential difference is measured across each rod, DV I is not
affected by the internal resistance of the battery.

ii. 1 point

For indicating either: 1 point


The students should be concerned because a change in temperature causes a change in
the resistance or resistivity.
OR
The students should not be concerned because any change in resistivity as the
temperature increases is small compared to measurement error.

© 2017 The College Board.


Visit the College Board on the Web: www.collegeboard.org.
AP® PHYSICS 2
2017 SCORING GUIDELINES

Question 3
12 points total Distribution
of points
(a) 2 points

For the arrow drawn upside down relative to the object 1 point
For bar/circle drawn left-to-right reversed relative to the object or consistent with an 1 point
(incorrect) upright arrow

(b)
i. 1 point

For correctly showing a calculation of the focal length and a correct answer with units 1 point
1 1 1 1 1
= + = +
f do di 20 cm 30 cm
f = 12 cm

ii. 1 point

For correctly showing a calculation of the magnitude of the magnification (with or 1 point
without sign) and a correct answer
M = d i d o = 30 cm 20 cm
M = 1.5

(c)
i. 1 point

For two reasonably correctly drawn rays consistent with the calculated focal length, and 1 point
inclusion of an inverted image

© 2017 The College Board.


Visit the College Board on the Web: www.collegeboard.org.
AP® PHYSICS 2
2017 SCORING GUIDELINES

Question 3 (continued)
Distribution
of points

(c) (continued)
ii. 2 points

For a correct explanation of how the rays drawn relate to the focal length 1 point
For a correct explanation of how the image relates to the magnification 1 point
Example: The horizontal ray from the object bends to cross the axis at 12 cm from the
middle of the lens, which is the focal length. The image arrow is about 1.5 times the
height of the object, which is the magnification.

(d)
i. 2 points

For showing the downward refraction of the rays at each surface of the lens in air (i.e., 1 point
toward the normal entering the lens and away from the normal leaving the lens)
For the rays refracted by the lens in water at greater angles to the normal than the 1 point
corresponding angles for the lens in air — i.e., less bending (can be earned even if
first point was not, scoring is relative to whatever is drawn for the lens in air)

ii. 3 points

For describing a greater focal length when the lens is in water or a comparison 1 point
consistent with part (d)(i)
For describing a larger image distance and image size or a comparison consistent with 1 point
part (d)(i)
For describing how the rays drawn in (d)(i) support the descriptions 1 point
Example: The rays do not bend as much as they pass from water to glass as when they
pass from air to glass (and vice versa). This means parallel rays coming into the lens
will converge at a farther distance, so the focal length is longer. Rays from an object
also will converge farther from the lens, so the new image is farther and larger.

© 2017 The College Board.


Visit the College Board on the Web: www.collegeboard.org.
AP® PHYSICS 2
2017 SCORING GUIDELINES

Question 4
10 points total Distribution
of points
(a) 1 point

For an arrow pointing outward from the object, along a diagonal of the square and away 1 point
from the object with charge -2Q , with no other arrows

(b)
i. 3 points

For correctly determining magnitudes of the field from individual objects 1 point
The fields from the +2Q objects cancel. This can be implicit or explicit in the
calculations.
For the -2Q object, E = 2 kQ d 2
For the +Q object, E = kQ d 2
For correctly adding the individual fields 1 point
E = 3kQ d 2
For showing the correct direction on the diagram, along a diagonal of the square and 1 point
toward the object with charge -2Q

ii. 1 point

For showing a correct scalar potential summation 1 point


A final answer is not required; however, no credit is given if an incorrect final answer is
included.
V   k d  2Q  Q  2Q  2Q   3kQ d

© 2017 The College Board.


Visit the College Board on the Web: www.collegeboard.org.
AP® PHYSICS 2
2017 SCORING GUIDELINES

Question 4 (continued)
Distribution
of points

(c) 5 points

For indicating that electric potential energy is the energy stored in a configuration of 1 point
charged objects
For indicating that the change in potential energy is equal to the work done by an 1 point
external force to create a particular configuration
For indicating that moving the object with +2Q charge results in an increase in energy 1 point
and indicating that moving the object with +Q charge results in a decrease in
energy (i.e., for showing understanding that moving charges of the same sign closer
together increases the energy and/or moving charges of opposite sign closer together
decreases the energy, with some support such as U = kqQ r or a description of
doing work against forces)
For indicating that the net result is an increase in the energy with some explanation 1 point
For a logical, relevant, and internally consistent response that addresses the required 1 point
argument or question asked, and follows the guidelines described in the published
requirements for the paragraph-length response

© 2017 The College Board.


Visit the College Board on the Web: www.collegeboard.org.
2018

AP Physics 2:
Algebra-Based
Scoring Guidelines

© 2018 The College Board. College Board, Advanced Placement Program, AP, AP Central, and the acorn logo
are registered trademarks of the College Board. Visit the College Board on the Web: www.collegeboard.org.
AP Central is the official online home for the AP Program: apcentral.collegeboard.org
AP® PHYSICS
2018 SCORING GUIDELINES

General Notes About 2018 AP Physics Scoring Guidelines

1. The solutions contain the most common method of solving the free-response questions and the allocation of
points for this solution. Some also contain a common alternate solution. Other methods of solution also
receive appropriate credit for correct work.

2. The requirements that have been established for the paragraph-length response in Physics 1 and Physics 2 can
be found on AP Central at
https://secure-media.collegeboard.org/digitalServices/pdf/ap/paragraph-length-response.pdf.

3. Generally, double penalty for errors is avoided. For example, if an incorrect answer to part (a) is correctly
substituted into an otherwise correct solution to part (b), full credit will usually be awarded. One exception to
this may be cases when the numerical answer to a later part should be easily recognized as wrong, e.g., a
speed faster than the speed of light in vacuum.

4. Implicit statements of concepts normally receive credit. For example, if use of the equation expressing a
particular concept is worth 1 point, and a student’s solution embeds the application of that equation to the
problem in other work, the point is still awarded. However, when students are asked to derive an expression,
it is normally expected that they will begin by writing one or more fundamental equations, such as those
given on the exam equation sheet. For a description of the use of such terms as “derive” and “calculate” on
the exams, and what is expected for each, see “The Free-Response Sections  Student Presentation” in the
AP Physics; Physics C: Mechanics, Physics C: Electricity and Magnetism Course Description or “Terms
Defined” in the AP Physics 1: Algebra-Based Course and Exam Description and the AP Physics 2: Algebra-
Based Course and Exam Description.

5. The scoring guidelines typically show numerical results using the value g  9.8 m s 2 , but the use of
10 m s 2 is of course also acceptable. Solutions usually show numerical answers using both values when they
are significantly different.

6. Strict rules regarding significant digits are usually not applied to numerical answers. However, in some cases
answers containing too many digits may be penalized. In general, two to four significant digits are acceptable.
Numerical answers that differ from the published answer due to differences in rounding throughout the
question typically receive full credit. Exceptions to these guidelines usually occur when rounding makes a
difference in obtaining a reasonable answer. For example, suppose a solution requires subtracting two
numbers that should have five significant figures and that differ starting with the fourth digit (e.g., 20.295 and
20.278). Rounding to three digits will lose the accuracy required to determine the difference in the numbers,
and some credit may be lost.

© 2018 The College Board.


Visit the College Board on the Web: www.collegeboard.org.
AP® PHYSICS 2
2018 SCORING GUIDELINES

Question 1
10 points total Distribution
of points

The figures above show a rectangular conducting loop at three instants in time. The loop moves at a constant
speed v into and through a region of constant, uniform magnetic field B directed into the page. The magnetic
field is zero outside the region.

(a) LO 2.D.1.1, SP 2.2; LO 4.E.2.1, SP 6.4


5 points

In a coherent paragraph-length response, compare the magnitude and direction of the current at times t1 , t2 ,
and t3 . Include an explanation of why there is or is not a current and the direction of the current if one is
present. Use fundamental physics concepts and principles in your explanation.

For indicating that the currents at t1 and t2 have equal nonzero magnitudes and are in 1 point
the same direction
For indicating that there is no current at t3 1 point
For correctly indicating that the currents depend on the change in flux through the loop 1 point
or the forces on the charges moving in the field
For correctly identifying the direction of the current as counter-clockwise and either 1 point
explaining that the direction of the current generates a magnetic field that opposes
the change in flux or analyzing the force on the charge carriers in each segment of
the loop
For an on-topic response that has sufficient paragraph structure, as described in the 1 point
published requirements for the paragraph length response

(b)
The loop is removed. A proton traveling to the right in the plane of the page, as shown below, then enters the
region of magnetic field with a speed v = 3.0 ¥ 10 5 m s . The magnitude of the field is 0.030 T. The effects
of gravity are negligible.

© 2018 The College Board.


Visit the College Board on the Web: www.collegeboard.org.
AP® PHYSICS 2
2018 SCORING GUIDELINES

Question 1 (continued)
Distribution
of points

(b) (continued)

i. LO 2.D.1.1, SP 2.2
1 point

Calculate the magnitude of the force on the proton as it enters the field.

For correct substitutions into a correct expression and correct units on the final answer 1 point
F = qvB
( )( )
F = 1.6 ¥ 10 -19 C 3.0 ¥ 10 5 m s ( 0.03 T )

F = 1.4 ¥ 10 -15 N

ii. LO 2.D.1.1, SP 2.2; LO 3.B.1.4, SP 6.4; LO 3.C.3.1, SP 1.4


1 point

On the figure below, sketch a possible path of the proton as it travels through the magnetic field. Clearly label
the path P1.

P1 P2

For drawing a curved arc through the field, curved upward where the proton enters 1 point
Anything greater than a semi-circle or a path that does not reach the edge of the field
does not earn credit. Any path after exiting the field is ignored.

iii. LO 2.D.1.1, SP 2.2; LO 3.B.1.4, SP 6.4


1 point

A second proton now enters the magnetic field at the same point and from the same direction but at a greater
speed than the first proton. On the figure above, draw the path of the second proton as it travels through the
field. Clearly label the path P2.

For drawing a path with a larger radius that is consistent with answer to (b)(ii) 1 point

© 2018 The College Board.


Visit the College Board on the Web: www.collegeboard.org.
AP® PHYSICS 2
2018 SCORING GUIDELINES

Question 1 (continued)
Distribution
of points

(b) (continued)

iv. LO 2.C.1.1, SP 6.4; LO 2.C.1.2, SP 2.2; LO 3.B.2.1, SP 1.4, 2.2


2 points

Next an electric field is applied in the same region as the magnetic field, such that there is no net force on the
first proton as it enters the region. Calculate the magnitude and indicate the direction of the electric field
relative to the coordinate system shown in part (b).

For indicating a direction of the electric field that is consistent with the response to 1 point
(b)(ii)
Given the correct response to (b)(ii) illustrated above, the electric field must be directed
in the -y direction (or toward the bottom of the page)
For equating the electric and magnetic forces and substituting into the correct expression 1 point
using values consistent with the response to (b)(i)
qE = qvB (Implicitly equating the calculated magnetic force to the electric force is
acceptable.)
E = vB
( )
E = 3.0 ¥ 10 5 m s ( 0.03 T )
E = 9000 N C

Learning Objectives (LO)

LO 2.C.1.1: The student is able to predict the direction and the magnitude of the force exerted on an object with
an electric charge q placed in an electric field E using the mathematical model of the relation between an
 
electric force and an electric field: F = qE ; a vector relation. [See Science Practices 6.4, 7.2]
LO 2.C.1.2: The student is able to calculate any one of the variables — electric force, electric charge, and electric
field — at a point given the values and sign or direction of the other two quantities. [See Science Practice 2.2]
LO 2.D.1.1: The student is able to apply mathematical routines to express the force exerted on a moving charged
object by a magnetic field. [See Science Practice 2.2]
LO 3.B.1.4: The student is able to predict the motion of an object subject to forces exerted by several objects
using an application of Newton’s second law in a variety of physical situations. [See Science Practices 6.4,
7.2]
LO 3.B.2.1: The student is able to create and use free-body diagrams to analyze physical situations to solve
problems with motion qualitatively and quantitatively. [See Science Practices 1.1, 1.4, 2.2]
LO 3.C.3.1: The student is able to use right-hand rules to analyze a situation involving a current-carrying
conductor and a moving electrically charged object to determine the direction of the magnetic force exerted
on the charged object due to the magnetic field created by the current-carrying conductor. [See Science
Practice 1.4]
LO 4.E.2.1: The student is able to construct an explanation of the function of a simple electromagnetic device in
which an induced emf is produced by a changing magnetic flux through an area defined by a current loop
(i.e., a simple microphone or generator) or of the effect on behavior of a device in which an induced emf is
produced by a constant magnetic field through a changing area. [See Science Practice 6.4]

© 2018 The College Board.


Visit the College Board on the Web: www.collegeboard.org.
AP® PHYSICS 2
2018 SCORING GUIDELINES

Question 2
12 points total Distribution
of points

Students are given resistor 1 with resistance R1 connected in series with the parallel combination of a switch
S and resistor 2 with resistance R2 , as shown above. The circuit elements cannot be disconnected from each
other, and other circuit components can only be connected at points A and B. The students also are given an
ammeter and one 9 V battery. The teacher instructs the students to take measurements that can be used to
determine R1 and R2 .

(a) LO 4.E.5.3, SP 2.2, 4.2, 5.1; LO 5.B.9.5, SP 6.4; LO 5.C.3.4, SP 6.4


4 points

Complete the diagram below to show how the ammeter and the battery should be connected to experimentally
determine the resistance of each resistor. Describe the experiment by listing the measurements to be taken and
explaining how the measurements would be used to calculate resistances R1 and R2 .

For a diagram with an ammeter and battery in series with the resistor combination 1 point
For indicating that the current should be measured with the switch closed and open 1 point
For correctly indicating that with the switch closed R1  V I1 1 point
For correctly indicating that with the switch open
 R2 V I 2   R1 1 point

A second group of students is given a combination of circuit elements that is similar to the previous one but
has an initially uncharged capacitor in series with the open switch, as shown above. The combination is
placed in a circuit with a power supply so that the potential difference between A and B is maintained at 9 V.
The students close the switch and immediately begin to record the current through point B. The initial current
is 0.9 A, and after a long time the current is 0.3 A.

© 2018 The College Board.


Visit the College Board on the Web: www.collegeboard.org.
AP® PHYSICS 2
2018 SCORING GUIDELINES

Question 2 (continued)
Distribution
of points
(b)
i. LO 4.E.5.2, SP 6.1, 6.4; LO 5.B.9.5, SP 6.4; LO 5.C.3.7, SP 1.4
3 points

Compare the currents through resistor 1, resistor 2, and the switch immediately after the switch is closed to
the currents a long time after the switch is closed. Specifically state if any current is zero.

For indicating that the current through resistor 1 immediately after the switch is closed is 1 point
greater than the current a long time after the switch is closed
For indicating that the current through resistor 2 is zero immediately after the switch is 1 point
closed and nonzero a long time after the switch is closed
For indicating that the current through the switch is nonzero immediately after the 1 point
switch is closed and zero a long time after the switch is closed

ii. LO 4.E.5.1, SP 2.2, 6.4


2 points

Calculate the values of R1 and R2 .

For using the correct value of current and correctly calculating R1 1 point
9 V   0.9 A  R1
R1  10 W
For using the correct value of current and correctly calculating R2 , consistent with the 1 point
calculated value of R1

9V 0.3 A   R1 
R2   0.3 A  10 W  R2 
R2  20 W

iii. LO 4.E.5.1, SP 2.2; LO 5.B.9.6, SP 2.2, LO 5.C.3.7, SP 1.4, 2.2


1 point

Determine the potential difference across the capacitor a long time after the switch is closed.

For correctly calculating the potential difference across the capacitor, including correct 1 point
units, consistent with part (b)(ii)
VC  9 V   0.3 A 10 W
Vbattery  Vresistor 1 
VC  6 V

© 2018 The College Board.


Visit the College Board on the Web: www.collegeboard.org.
AP® PHYSICS 2
2018 SCORING GUIDELINES

Question 2 (continued)
Distribution
of points
A third group of students now uses the combination of circuit elements with the capacitor. They connect it to
a 9 V battery that they treat as ideal but which is actually not ideal and has internal resistance.

(c) LO 5.B.9.7, SP 5.3


2 points

How does the third group’s value of R1 calculated from the data they collected compare to the second group’s
value? Explain your reasoning with reference to physics principles and/or mathematical models.

For correctly explaining that the third group’s measured current is smaller 1 point
For correctly indicating that the third group’s value of R1 is higher than the second 1 point
group’s or the resistance they will determine is actually R1  r

Learning Objectives (LO)

LO 4.E.5.1: The student is able to make and justify a quantitative prediction of the effect of a change in values or
arrangements of one or two circuit elements on the currents and potential differences in a circuit containing a
small number of sources of emf, resistors, capacitors, and switches in series and/or parallel. [See Science
Practices 2.2, 6.4]
LO 4.E.5.2: The student is able to make and justify a qualitative prediction of the effect of a change in values or
arrangements of one or two circuit elements on currents and potential differences in a circuit containing a
small number of sources of emf, resistors, capacitors, and switches in series and/or parallel. [See Science
Practices 6.1, 6.4]
LO 4.E.5.3: The student is able to plan data collection strategies and perform data analysis to examine the values
of currents and potential differences in an electric circuit that is modified by changing or rearranging circuit
elements, including sources of emf, resistors, and capacitors. [See Science Practices 2.2, 4.2, 5.1]
LO 5.B.9.5: The student is able to use conservation of energy principles (Kirchhoff’s loop rule) to describe and
make predictions regarding electrical potential difference, charge, and current in steady-state circuits
composed of various combinations of resistors and capacitors. [See Science Practice 6.4]
LO 5.B.9.6: The student is able to mathematically express the changes in electric potential energy of a loop in a
multiloop electrical circuit and justify this expression using the principle of the conservation of energy. [See
Science Practices 2.1, 2.2]
LO 5.B.9.7: The student is able to refine and analyze a scientific question for an experiment using Kirchhoff’s
loop rule for circuits that includes determination of internal resistance of the battery and analysis of a non-
ohmic resistor. [See Science Practices 4.1, 4.2, 5.1, 5.3]
LO 5.C.3.4: The student is able to predict or explain current values in series and parallel arrangements of resistors
and other branching circuits using Kirchhoff’s junction rule and relate the rule to the law of charge
conservation. [See Science Practices 6.4, 7.2]
LO 5.C.3.7: The student is able to determine missing values, direction of electric current, charge of capacitors at
steady state, and potential differences within a circuit with resistors and capacitors from values and directions
of current in other branches of the circuit. [See Science Practice 1.4, 2.2]

© 2018 The College Board.


Visit the College Board on the Web: www.collegeboard.org.
AP® PHYSICS 2
2018 SCORING GUIDELINES

Question 3
12 points total Distribution
of points

Monochromatic light of frequency f shines on a metal, as shown above. The frequency of the light is varied,
and for some frequencies electrons are emitted from the metal. The maximum kinetic energy K e of the
emitted electrons is measured as a function of the frequency of the light.

(a)
i. LO 5.B.4.2, SP 1.4, 2.1, 2.2; LO 6.F.3.1, SP 6.4
3 points

Based on conservation of energy, the relationship between K e and f is predicted to be Af B  K e when
f  f0 and K e  0 when f  f0 , where A and B are positive constants. A graph of this relationship is
shown below. Indicate which aspects of the graph correspond to A and B. Also, explain the physical
meaning of A, B, and f0 .

For indicating that A represents the slope or the rate of change of K e as a function of f 1 point
and equals Planck’s constant
For indicating that -B is the intercept with the K e axis and equals the minimum energy 1 point
needed to release an electron from the metal (the work function)
For indicating that f0 is the minimum frequency that will release an electron from the 1 point
metal (the cutoff or threshold frequency)

ii. LO 6.F.3.1, SP 6.4


1 point

Explain the physical meaning of the horizontal section of the graph between the origin and f0 .

For indicating that the horizontal portion of the graph represents frequencies of light 1 point
whose energy is insufficient to eject an electron

© 2018 The College Board.


Visit the College Board on the Web: www.collegeboard.org.
AP® PHYSICS 2
2018 SCORING GUIDELINES

Question 3 (continued)
Distribution
of points
(a) (continued)

iii. LO 6.F.3.1, SP 6.4


3 points

A second metal with different properties than the first metal is now used. On the figure below, the dashed
lines are the same lines shown in the previous graph. Sketch lines on the figure below that could represent
the data for the second metal. Explain one difference between the two graphs.

For drawing a line that is parallel to the given line 1 point


For drawing the horizontal intercept on either side of f0 with the line ending at the 1 point
horizontal axis (The horizontal segment does not have to be drawn.)
For indicating that the K e or f intercept changes because the work function or the 1 point
frequency at which electrons can be emitted is different

(b)
The figure below shows an electroscope. A sphere is connected by a vertical bar to the leaves, which are
thin, light strips of material. The sphere, leaves, and bar are all made of metal. The electroscope initially
has a negative charge, so the leaves are separated.

i. LO 1.B.1.2, SP 6.4, 7.2, LO 4.E.3.3, SP 6.4; LO 6.F.3.1, SP 6.4


2 points

Ultraviolet (UV) light shines on the sphere, causing the leaves of the electroscope to move closer together.
Explain why this happens.

For indicating that the UV light causes electrons to be ejected from the electroscope 1 point
For indicating that the electroscope becomes less negatively charged, causing the leaves 1 point
to move closer together

© 2018 The College Board.


Visit the College Board on the Web: www.collegeboard.org.
AP® PHYSICS 2
2018 SCORING GUIDELINES

Question 3 (continued)
Distribution
of points
(b) (continued)

ii. LO 6.F.1.1, SP 6.4, 7.2, LO 6.F.3.1, SP 6.4


1 point

Green light then shines on an identical negatively charged electroscope. No movement of the leaves is
observed. Explain why the green light does not make the leaves move, while the UV light does.

For indicating that the green light frequency or energy per photon is too low to eject 1 point
electrons

(c) LO 6.F.3.1, SP 6.4


2 points

The brightness of the green light is increased until the intensity (power per unit area) is the same as that of
the UV light. What aspect of the green light changes when its brightness is increased? Would shining the
brighter green light on the electroscope result in movement of the leaves? Explain why or why not.

For indicating that the increase in brightness causes an increase in the number of 1 point
photons in the beam or increases the amplitude of the wave
For indicating that the leaves would not separate because the energy per photon or 1 point
frequency of the light remains the same
The particle nature of light (photons) must be discussed to receive full credit.

Learning Objectives (LO)

LO 1.B.1.2: The student is able to make predictions, using the conservation of electric charge, about the sign
and relative quantity of net charge of objects or systems after various charging processes, including
conservation of charge in simple circuits. [See Science Practices 6.4, 7.2]
LO 4.E.3.3: The student is able to construct a representation of the distribution of fixed and mobile charge in
insulators and conductors. [See Science Practices 1.1, 1.4, 6.4]
LO 5.B.4.2: The student is able to calculate changes in kinetic energy and potential energy of a system, using
information from representations of that system. [See Science Practices 1.4, 2.1, 2.2]
LO 6.F.1.1: The student is able to make qualitative comparisons of the wavelengths of types of electromagnetic
radiation. [See Science Practices 6.4, 7.2]
LO 6.F.3.1: The student is able to support the photon model of radiant energy with evidence provided by the
photoelectric effect. [See Science Practice 6.4]

© 2018 The College Board.


Visit the College Board on the Web: www.collegeboard.org.
AP® PHYSICS 2
2018 SCORING GUIDELINES

Question 4
10 points total Distribution
of points

A large boat like the one shown above has a mass M b and can displace a maximum volume Vb . The boat is
floating in a river with water of density rwater and is being loaded with steel beams each of density rsteel and
volume Vsteel . The boat owners want to be able to carry as many beams as possible.

(a) LO 1.E.1.2, SP 6.4; LO 3.B.2.1, SP 1.1, 1.4, 2.2; LO 5.B.10.1, SP 2.2


4 points

Derive an expression for the maximum number N of steel beams that can be loaded on the boat without
exceeding the maximum displaced volume, in terms of the given quantities and physical constants, as
appropriate.

For equating the correct forces acting on the boat-steel system: gravity (weight) and the 1 point
buoyant force
For correctly calculating the weight of the boat-steel system 1 point
  M b  N steel rsteelVsteel  g , where N is the number of steel beams (must clearly
Wsystem
use mass of boat)
For correctly calculating the buoyant force 1 point
Fb  rwater gVb
For algebraic manipulation of the equations to get an expression for the number of 1 point
beams consistent with the equations for weight and buoyant force
 Mb  N rsteelVsteel  g 
rwater gVb
N  rwaterVb  M b  rsteelVsteel

(b) LO 6.C.1.1, SP 6.4, 7.2; LO 6.E.1.1, SP 6.4, 7.2; LO 6.E.3.3, SP 6.4, 7.2
4 points

The captain realizes that oil is leaking from the boat, creating a thin film of oil on the water surface. In
one area of the oil film the surface looks mostly green. Explain in detail how constructive interference
contributes to the green appearance. Assume the index of refraction of the oil is greater than the index of
refraction of the water.

The constructive interference is between light reflected from the air-oil interface and
light reflected from the oil-water interface.
For indicating that the green appearance is the result of interference of light from two 1 point
waves
For indicating that there is a phase shift due to one of the reflections 1 point
For indicating that the wavelength of the light is different in air and oil 1 point
For indicating that there is a path-length difference of the light reflected from the two 1 point
surfaces

© 2018 The College Board.


Visit the College Board on the Web: www.collegeboard.org.
AP® PHYSICS 2
2018 SCORING GUIDELINES

Question 4 (continued)
Distribution
of points

(c) LO 5.F.1.1, SP 2.2, 7.2


2 points

Later the boat is floating down the river with the water current, heading for a town. The river has a width of
60 m and a constant depth and flows at a speed of 5 km hr . Partway to the town, the river narrows to a width
of 30 m while its depth remains the same. Calculate the speed of the water in the narrow section.

For an attempting to apply the principle of continuity 1 point


Awide vwide  Anarrow vnarrow
60 m depth 5 km hr   30 m depth  vnarrow 
For correctly calculating the speed 1 point
vnarrow  10 km hr

Learning Objectives (LO)

LO 1.E.1.2: The student is able to select from experimental data the information necessary to determine the
density of an object and/or compare densities of several objects. [See Science Practices 4.1, 6.4]
LO 3.B.2.1: The student is able to create and use free-body diagrams to analyze physical situations to solve
problems with motion qualitatively and quantitatively. [See Science Practices 1.1, 1.4, 2.2]
LO 5.B.10.1: The student is able to use Bernoulli’s equation to make calculations related to a moving fluid. [See
Science Practice 2.2]
LO 5.F.1.1: The student is able to make calculations of quantities related to flow of a fluid, using mass
conservation principles (the continuity equation). [See Science Practices 2.1, 2.2, 7.2]
LO 6.C.1.1: The student is able to make claims and predictions about the net disturbance that occurs when two
waves overlap. Examples should include standing waves. [See Science Practices 6.4, 7.2]
LO 6.E.1.1: The student is able to make claims using connections across concepts about the behavior of light as
the wave travels from one medium into another, as some is transmitted, some is reflected, and some is
absorbed. [See Science Practices 6.4, 7.2]
LO 6.E.3.3: The student is able to make claims and predictions about path changes for light traveling across a
boundary from one transparent material to another at non-normal angles resulting from changes in the speed
of propagation. [See Science Practices 6.4, 7.2]

© 2018 The College Board.


Visit the College Board on the Web: www.collegeboard.org.
2019

AP Physics 2:
®

Algebra-Based
Scoring Guidelines

© 2019 The College Board. College Board, Advanced Placement, AP, AP Central, and the acorn logo are
registered trademarks of the College Board. Visit the College Board on the web: collegeboard.org.
AP Central is the official online home for the AP Program: apcentral.collegeboard.org.
AP® PHYSICS
2019 SCORING GUIDELINES

General Notes About 2019 AP Physics Scoring Guidelines

1. The solutions contain the most common method of solving the free-response questions and the allocation of
points for this solution. Some also contain a common alternate solution. Other methods of solution also
receive appropriate credit for correct work.

2. The requirements that have been established for the paragraph-length response in Physics 1 and Physics 2 can
be found on AP Central at
https://secure-media.collegeboard.org/digitalServices/pdf/ap/paragraph-length-response.pdf.

3. Generally, double penalty for errors is avoided. For example, if an incorrect answer to part (a) is correctly
substituted into an otherwise correct solution to part (b), full credit will usually be awarded. One exception to
this may be cases when the numerical answer to a later part should be easily recognized as wrong, e.g., a
speed faster than the speed of light in vacuum.

4. Implicit statements of concepts normally receive credit. For example, if use of the equation expressing a
particular concept is worth 1 point, and a student’s solution embeds the application of that equation to the
problem in other work, the point is still awarded. However, when students are asked to derive an expression,
it is normally expected that they will begin by writing one or more fundamental equations, such as those
given on the exam equation sheet. For a description of the use of such terms as “derive” and “calculate” on
the exams, and what is expected for each, see “The Free-Response Sections  Student Presentation” in the
AP Physics; Physics C: Mechanics, Physics C: Electricity and Magnetism Course Description or “Terms
Defined” in the AP Physics 1: Algebra-Based Course and Exam Description and the AP Physics 2: Algebra-
Based Course and Exam Description.

5. The scoring guidelines typically show numerical results using the value g = 9.8 m s 2 , but the use of
10 m s 2 is of course also acceptable. Solutions usually show numerical answers using both values when they
are significantly different.

6. Strict rules regarding significant digits are usually not applied to numerical answers. However, in some cases
answers containing too many digits may be penalized. In general, two to four significant digits are acceptable.
Numerical answers that differ from the published answer due to differences in rounding throughout the
question typically receive full credit. Exceptions to these guidelines usually occur when rounding makes a
difference in obtaining a reasonable answer. For example, suppose a solution requires subtracting two
numbers that should have five significant figures and that differ starting with the fourth digit (e.g., 20.295 and
20.278). Rounding to three digits will lose the accuracy required to determine the difference in the numbers,
and some credit may be lost.

© 2019 The College Board.


Visit the College Board on the web: collegeboard.org.
AP® PHYSICS 2
2019 SCORING GUIDELINES

Question 1

10 points

The figure above shows a particle with positive charge +Q traveling with a constant speed v0 to the right and
in the plane of the page. The particle is approaching a region, shown by the dashed box, that contains a
constant uniform field. The effects of gravity are negligible.

(a)
i. LO 2.C.1.1, SP 6.4
2 points

On the figure below, draw a possible path of the particle in the region if the region contains only an electric
field directed toward the bottom of the page.

For a curved path that is initially horizontal and does not have a component of velocity 1 point
toward the left
For a path that deflects toward the bottom of the page and reaches an edge of the region 1 point

ii. LO 3.C.3.1, SP 1.4


2 points

On the figure below, draw a possible path of the particle in the region if the region contains only a magnetic
field directed out of the page.

For a curved path that is initially horizontal, is not more than a semicircle, and reaches 1 point
an edge of the region
For a path that deflects toward the bottom of the page 1 point

© 2019 The College Board.


Visit the College Board on the web: collegeboard.org.
AP® PHYSICS 2
2019 SCORING GUIDELINES

Question 1 (continued)

(a) (continued)

iii. LO 2.C.5.3, SP 1.1, 7.1


1 point

For which of the previous situations is the motion more similar to that of a projectile in only a gravitational
field near Earth’s surface, and why?

For indicating that the motion in the electric field is more similar to a projectile because 1 point
the force or acceleration is always down or constant, or the shape is parabolic

(b) LO 2.D.1.1, SP 2.2; LO 3.A.3.4, SP 6.1, 6.4; LO 3.B.1.4, SP 6.4, 7.2; LO 3.B.2.1, SP 1.1, 1.4, 2.2
5 points

Another region of space contains an electric field directed toward the top of the page and a magnetic field
directed out of the page. Both fields are constant and uniform. A horizontal beam of protons with a
variety of speeds enters the region, as shown above. Protons exit the region at a variety of locations,
including points 1 and 2 shown on the figure. In a coherent, paragraph-length response, explain why some
protons exit the region at point 1 and others exit at point 2. Use physics principles to explain your
reasoning.

For indicating that initially the electric and magnetic forces act in opposite directions 1 point
For indicating or implying that the magnetic force is affected by speed, but the electric 1 point
force is not
For indicating that different paths occur as a result of the addition of forces 1 point
For indicating that slower protons exit higher than faster protons (i.e., slower protons 1 point
exit at point 1 and faster protons exit at point 2)
For a logical, relevant, and internally consistent argument that addresses the question 1 point
asked and follows the guidelines described in the published requirements for the
paragraph-length response
Example:

For a charged particle to travel through the region undeflected, the net force on it must
be zero. This means that the upward electric force and the downward magnetic force
must be equal and opposite to each other. This occurs for a particular speed. The
electric force is independent of the particle’s velocity, but the magnetic force will be
larger for greater velocities and less for smaller velocities. If a particle is moving
faster than the particular speed, it will experience a greater magnetic force and be
deflected downward. If it is moving more slowly than the particular speed, it will be
deflected upward.

© 2019 The College Board.


Visit the College Board on the web: collegeboard.org.
AP® PHYSICS 2
2019 SCORING GUIDELINES

Question 1 (continued)

(b) (continued)

Claim: Slower protons exit higher than faster protons (i.e., slower protons exit at point 1
and faster protons exit at point 2).
Evidence: The electric and magnetic forces act in opposite directions. The magnetic
force is affected by speed, but the electric force is not.
Reasoning: Different paths occur as a result of the addition of forces.

Learning Objectives

LO 2.C.1.1: The student is able to predict the direction, and the magnitude of the force exerted on an object with
an electric charge q placed in an electric field E using the mathematical model of the relation between an
 
electric force and an electric field: F  qE ; a vector relation. [See Science Practices 6.4, 7.2]
LO 2.C.5.3: The student is able to represent the motion of an electrically-charged particle in the uniform field
between two oppositely charged plates and express the connection of this motion to projectile motion of an
object with mass in Earth’s gravitational field. [See Science Practices 1.1, 2.2, 7.1]
LO 2.D.1.1: The student is able to apply mathematical routines to express the force exerted on a moving charged
object by a magnetic field. [See Science Practices 2.2]
LO 3.A.3.4: The student is able to make claims about the force on an object due to the presence of other objects
with the same property: mass, electric charge. [See Science Practices 6.1, 6.4]
LO 3.B.1.4: The student is able to predict the motion of an object subject to forces exerted by several objects
using an application of Newton’s second law in a variety of physical situations. [See Science Practices 6.4,
7.2]
LO 3.B.2.1: The student is able to create and use free-body diagrams to analyze physical situations to solve
problems with motion qualitatively and quantitatively. [See Science Practices 1.1, 1.4, 2.2]
LO 3.C.3.1: The student is able to use right-hand rules to analyze a situation involving a current-carrying
conductor and a moving electrically charged object to determine the direction of the magnetic force exerted
on the charged object due to the magnetic field created by the current-carrying conductor. [See Science
Practices 1.4]

© 2019 The College Board.


Visit the College Board on the web: collegeboard.org.
AP® PHYSICS 2
2019 SCORING GUIDELINES

Question 2

12 points

The two circuits shown above contain an ideal variable power supply, an ohmic resistor of resistance R, an
ammeter A, and two voltmeters VPS and VR . In circuit 1 the ammeter has negligible resistance, and in
circuit 2 the ammeter has significant internal ohmic resistance r. The potential difference of the power supply
is varied, and measurements of current and potential difference are recorded.

(a) LO 4.E.5.1, SP 6.4


2 points

The axes below can be used to graph the current measured by the ammeter as a function of the potential
difference measured across the power supply. On the axes, do the following.
 Sketch a possible graph for circuit 1 and label it 1.
 Sketch a possible graph for circuit 2 and label it 2.

For graph 1 a straight line with a positive slope through origin 1 point
For graph 2 a straight line with a positive slope through origin with a smaller slope than 1 point
line 1

(b) LO 5.B.9.6, SP 2.2; LO 5.C.3.4, SP 6.4


2 points

Let VPS be the potential difference measured by voltmeter VPS across the power supply, and let I be the
current measured by the ammeter A. For each circuit, write an equation that satisfies conservation of
energy, in terms of VPS , I, R, and r, as appropriate.
Circuit 1 Circuit 2
For a correct equation for circuit 1 1 point
VPS  IR  0
For a correct equation for circuit 2 1 point
VPS  I  R  r   0

© 2019 The College Board.


Visit the College Board on the web: collegeboard.org.
AP® PHYSICS 2
2019 SCORING GUIDELINES

Question 2 (continued)

(c) LO 5.B.9.8, SP 1.5


2 points

Explain how your equations in part (b) account for any differences between graphs 1 and 2 in part (a).

For indicating that the slope is inversely proportional to the resistance 1 point
For explaining that the equations in part (b) show that a larger total resistance 1 point
corresponds to a smaller slope or smaller current
Example:

Claim: The equations in part (b) account for the differences between graphs 1 and 2 in
part (a).
Evidence: The graphs show a linear relationship between current and potential
difference. The equations are linear functions, which when graphed would have a
slope that is the inverse of the total resistance.
Reasoning: The difference between the equations is the value of the total resistance, so
the equations account for the difference in slopes. The larger the total resistance, the
smaller the slope.

(d) LO 5.B.9.6, SP 2.2; LO 5.C.3.4, SP 6.4, 7.2


2 points

In circuit 2, R  40  . When voltmeter VPS reads 3.0 V, voltmeter VR reads 2.5 V. Calculate the
internal resistance r of the ammeter.

Ohm’s law solution:


For correctly calculating the current in the circuit 1 point
I  VR R  2.5 V 40   0.0625 A
For using Ohm’s law with the calculated current and correct potential difference 1 point
r  Vr I   3 V  2.5 V  0.0625 A
r 8

(e)
Voltmeter VR in circuit 2 is replaced by a resistor with resistance 120  to create circuit 3 shown below.
Voltmeter VPS still reads 3.0 V.

© 2019 The College Board.


Visit the College Board on the web: collegeboard.org.
AP® PHYSICS 2
2019 SCORING GUIDELINES

Question 2 (continued)

(e) (continued)
i. LO 4.E.5.1, SP 2.2
2 points

Calculate the equivalent resistance R eq of the circuit.

For calculating the equivalent resistance of the parallel branches 1 point


1 1 4
 
40  120  120 
R  30 
For adding the value of r from part (d) to R 1 point
Req  30   8   38 

ii. LO 5.B.9.6, SP 2.2


2 points

Calculate the current in each of the resistors that are in parallel.

For substituting the correct potential difference and the resistance from part (e)(i) into 1 point
Ohm’s law to determine the current through the battery
I tot  3 V 38   0.079 A
For calculating two currents that are in the correct ratio ( I 40   3 I120  ) 1 point
DVparallel = (3 V ) - (8 W)(0.079 A ) = 2.36 V
2.36 V
I 40    0.059 A
40 
2.36 V
I120    0.020 A
120 

Learning Objectives

LO 4.E.5.1: The student is able to make and justify a quantitative prediction of the effect of a change in values or
arrangements of one or two circuit elements on the currents and potential differences in a circuit containing a
small number of sources of emf, resistors, capacitors, and switches in series and/or parallel. [See Science
Practices 2.2, 6.4]
LO 5.B.9.6: The student is able to mathematically express the changes in electric potential energy of a loop in a
multiloop electrical circuit and justify this expression using the principle of the conservation of energy. [See
Science Practices 2.1, 2.2]
LO 5.B.9.8: The student is able to translate between graphical and symbolic representations of experimental data
describing relationships among power, current, and potential difference across a resistor. [See Science
Practices 1.5]
LO 5.C.3.4: The student is able to predict or describe current values in series and parallel arrangements of
resistors and other branching circuits using Kirchhoff’s junction rule and explain the relationship of the rule to
the law of charge conservation. [See Science Practices 6.4, 7.2]

© 2019 The College Board.


Visit the College Board on the web: collegeboard.org.
AP® PHYSICS 2
2019 SCORING GUIDELINES

Question 3

12 points

A group of students use the apparatus shown above to determine the thermal conductivity of a certain type of
plastic. A hot plate is used to keep water in a container boiling at a temperature of 100C . They place a slab
of the plastic with area 0.025 m 2 and thickness 0.010 m above the container so that the bottom surface of the
slab is at a temperature of 100C . They put a large block of ice with temperature 0C on top of the plastic
slab. Some of the ice melts, and the students measure the amount of water collected during a time t . The
students correctly calculate the amount of energy Q delivered to the ice and thus determine Q t . They
repeat this experiment several times, each time adding an identical slab to increase the total thickness L of
plastic. Their results are shown in the table below.

Table with sample entries for part (a)(ii)


Energy Flow Rate Q t  J s  97 53 31 27 18
Total Thickness of Plastic (m) 0.01 0.02 0.03 0.04 0.05
1/Thickness (1/m) 100 50 33.3 25 20

(a)
The students want to create a graph to yield a straight line whose slope could be used to calculate the thermal
conductivity of the plastic.
Sample graph using above data
Q/t
(J/s)
100

80

60

40

20

O 20 40 60 80 100
1/Thickness (1/m)

© 2019 The College Board.


Visit the College Board on the web: collegeboard.org.
AP® PHYSICS 2
2019 SCORING GUIDELINES

Question 3 (continued)

(a) (continued)

i. LO 1.E.3.1, SP 4.1, 5.1


1 point

Label the axes below to indicate a pair of quantities that could be graphed to yield a straight line. Include
units for the quantities.

Q kA T

t L
For labeling the axes with two quantities that would produce a linear graph, including 1 point
units
Example: Q t and 1 thickness

ii. LO 1.E.3.1, SP 4.1, 5.1


3 points

On the grid on the previous page, create a linear graph using the values for the quantities indicated in
part (a)(i). Be sure to do the following:
 Add to the data table the values of any quantities to be plotted that are not already given.
 Scale the axes.
 Plot the data from the table.
 Draw a line that best represents the data.

For scaling the axes linearly so the data extends over at least half of each axis 1 point
For accurately plotting the data 1 point
For a best-fit curve or line that fits the trend in the data 1 point

iii. LO 1.E.3.1, 5.1


2 points

Use the graph to calculate the thermal conductivity of the plastic.

For a correct method for calculating the slope using points on the best-fit line 1 point
80  20  J s 
For the graph above, slope   1.0 Jm s
80  20 1 m 
For determining the thermal conductivity k, with or without units using the slope found 1 point
above
Q kA T
 so slope  kA T
t L
 
Using slope above: k  slope A T  1 Jm s  0.025 m 100 C   0.40 J sm°C
2

© 2019 The College Board.


Visit the College Board on the web: collegeboard.org.
AP® PHYSICS 2
2019 SCORING GUIDELINES

Question 3 (continued)

(b) LO 5.B.6.1, SP1.2


2 points

Indicate one potential problem with the setup that could lead to an experimental value for the thermal
conductivity that is different from the actual value. Use physics principles to explain the effect this problem
could have on the experimental value.

For any valid indication of an additional thermal interaction with the environment 1 point
For a reasonable explanation of how additional energy added or lost could change the 1 point
experimental value of conductivity
Example 1: The given setup allows energy to be transferred to the ice from the air
around it. This means the values of Q t contain energy that did not go through the
plastic slab, resulting in a value of k that is too large.
Example 2: The given setup allows energy to be lost out the sides of the plastic slab.
This means the values of Q t do not contain all the energy that went through the
plastic slab, resulting in a value of k that is too small.
Claim: The problem leads to a value of k that is too small/large.
Evidence: The problem allows energy transfer into/out of the system that is not
accounted for.
Reasoning: The values of Q t contain less/more energy than went through the plastic
slab, resulting in a value of k that is too small/large.

(c) LO 4.C.3.1, SP 6.4


1 point

The rectangle below represents a side view of the plastic slab. Draw a single arrow on the diagram
representing the direction of the net flow of energy through the plastic.

For drawing an arrow toward the top of the page 1 point

(d) LO 4.C.3.1, SP 6.4; LO 5.B.6.1, SP 1.2; LO 5.D.1.6, SP 6.4


2 points

Describe what occurs in the plastic at the microscopic level that explains the energy flow you indicated in
part (c).

For indicating that particles at the bottom (or a location consistent with part (c)) have a 1 point
higher temperature or kinetic energy, so they vibrate faster
For indicating that particles collide with neighboring particles, transferring energy from 1 point
faster to slower particles in the process
Example: Energy absorbed at the lower surface makes particles jiggle faster, they jiggle
particles above them, and so forth until energy reaches the other side.

© 2019 The College Board.


Visit the College Board on the web: collegeboard.org.
AP® PHYSICS 2
2019 SCORING GUIDELINES

Question 3 (continued)
(e) LO 1.E.3.1, SP 4.1, 4.2
1 point

An extra plastic slab sits on a wood surface, with both the plastic slab and the wood surface at room
temperature. A student touches each and finds that the plastic slab feels cooler than the wood surface. Explain
what causes this observation.

For indicating that the slab and wood have different thermal conductivities or that 1 point
energy is transferred into the plastic and wood at different rates, with no incorrect
statements

Learning Objectives

LO 1.E.3.1: The student is able to design an experiment and analyze data from it to examine thermal
conductivity. [See Science Practices 4.1, 4.2, 5.1]
LO 4.C.3.1: The student is able to make predictions about the direction of energy transfer due to temperature
differences based on interactions at the microscopic level. [See Science Practices 6.4]
LO 5.B.6.1: The student is able to describe the models that represent processes by which energy can be
transferred between a system and its environment because of differences in temperature: conduction,
convection, and radiation. [See Science Practices 1.2]
LO 5.D.1.6: The student is able to make predictions of the dynamical properties of a system undergoing a
collision by application of the principle of linear momentum conservation and the principle of the
conservation of energy in situations in which an elastic collision may also be assumed. [See Science
Practices 6.4]

© 2019 The College Board.


Visit the College Board on the web: collegeboard.org.
AP® PHYSICS 2
2019 SCORING GUIDELINES

Question 4

10 points

A student notices many air bubbles rising through the water in a large fish tank at an aquarium.

(a) LO 6.E.3.1, SP 1.1, 1.4


3 points

In the figure below, the circle represents one such air bubble, and two incoming rays of light, A and B, are
shown. Ray B points toward the center of the circle. On the diagram, draw the paths of rays A and B as they
go through the bubble and back into the water. Your diagram should clearly show what happens to the rays at
each interface.

For ray B going straight through 1 point


For ray A bending away from the normal as it enters the air from the water 1 point
For ray A bending the opposite direction in relationship to the normal as it exits the air 1 point
and enters the water compared to the refraction entering the air from the water
Note: The normals need not be shown.

(b) LO 5.B.4.1, SP 6.4, 7.2; LO 5.B.4.2, SP 1.4, 7.2; LO 5.B.5.4, SP 6.4, 2.2; LO 5.B.5.5, SP 2.2, 6.4
3 points

The bubble has a volume V1 , the air inside it has density  A , and the water around it has density  W .
The bubble starts at rest and has a speed v f when it has risen a height h. Assume that the change in the
bubble’s volume is negligible. Derive an expression for the mechanical energy dissipated by drag forces
as the bubble rises this distance. Express your answer in terms of the given quantities and fundamental
constants, as appropriate.

For a valid application of the work-energy theorem 1 point


DK = Wnet = Wb - Wg - Wdiss
For finding the work done by the buoyant force 1 point
Wb = rwV1gh
For correct substitutions into an equation with consistent relative signs for the terms 1 point
1
r V v 2 = rW V1 gh - r A V1 gh - Wdiss
2 A 1 f
1
W diss = rW V1 gh - r A V1 gh - r A V1 v 2f
2

© 2019 The College Board.


Visit the College Board on the web: collegeboard.org.
AP® PHYSICS 2
2019 SCORING GUIDELINES

Question 4 (continued)

(c)

At a particular instant, one bubble is 4.5 m below the water’s surface. The surface of the water is at sea level,
and the density of the water is 1000 kg m3 .

i. LO 5.B.10.1, SP 2.2
1 point

Determine the absolute pressure in the bubble at this location.

P4.5m  Patm   w gd

   
P4.5m  1.0  105 Pa  1000 kg m3 9.8 m s2  4.5 m  
For a correct answer with units 1 point
P4.5m  1.44  10 Pa (or 1.45  10 Pa using g  10 m s )
5 5 2

ii. LO 7.A.3.3, SP 5.1


2 points

The bubble has a volume V1 when it is 4.5 m below the water’s surface. Assume that the temperature
of the air in the bubble remains constant as it rises. In terms of V1 , calculate the volume of the bubble
when it is just below the surface of the water.

For applying the ideal gas law at two locations in an attempt to determine the new 1 point
bubble volume
P4.5mV1  PatmVsurface
Vsurface  P4.5mV1 Patm
For substituting pressures consistent with part (i) 1 point
  
Vsurface  1.44  10 Pa V1 1  10 Pa
5 5

Vsurface  1.44V1 (or 1.45V1 using g  10 m s2 )

iii. LO 7.A.3.3, SP 5.1


1 point

If the air in the bubble cooled as it rose, the volume of the bubble would be less than the value
calculated in part (c)(ii). Use physics principles to briefly explain why.

For a correct explanation 1 point


Note: The explanation may be qualitative or quantitative. The explanation may also be
macroscopic or microscopic.

© 2019 The College Board.


Visit the College Board on the web: collegeboard.org.
AP® PHYSICS 2
2019 SCORING GUIDELINES

Question 4 (continued)
(c) (continued)
iii. (continued)

Example 1: By the ideal gas law, P4.5mV1 T1  PatmVsurface Tsurface , so


Vsurface  P4.5mV1Tsurface PatmT1 . The two pressures still have their previous values.
Tsurface  T1 , so the volume at the surface will be smaller.
Example 2: As the bubble cools, the air molecules move slower. Slower molecules exert
less force on the inner surface of the bubble. The unbalanced force, due to the
difference in the forces on the inside and outside of the bubble, causes the bubble to
expand less than it did in the constant temperature situation or contract.
Claim (given): The volume of the bubble will decrease

Example 1 evidence:
P4.5mV1 T1  PatmVsurface Tsurface , so Vsurface  P4.5mV1Tsurface PatmT1
Example 1 reasoning: The two pressures still have their previous values. Tsurface  T1 ,
so the volume at the surface will be smaller.

Example 2 evidence: As the bubble cools, the air molecules move slower. Slower
molecules exert less force on the inner surface of the bubble.
Example 2 reasoning: The unbalanced force, due to the difference in the forces on the
inside and outside of the bubble, causes the bubble to contract.

Learning Objectives:

LO 5.B.4.1: The student is able to describe and make predictions about the internal energy of systems. [See
Science Practices 6.4, 7.2]
LO 5.B.4.2: The student is able to calculate changes in kinetic energy and potential energy of a system using
information from representations of that system. [See Science Practices 1.4, 2.1, 2.2]
LO 5.B.5.4: The student is able to make claims about the interaction between a system and its environment in
which the environment exerts a force on the system, thus doing work on the system and changing the energy
of the system (kinetic energy plus potential energy). [See Science Practices 6.4, 7.2]
LO 5.B.5.5: The student is able to predict and calculate the energy transfer to (i.e., the work done on) an object or
system from information about a force exerted on the object or system through a distance. [See Science
Practices 2.2, 6.4]
LO 5.B.10.1: The student is able to make calculations related to a moving fluid using Bernoulli’s equation. [See
Science Practices 2.2]
LO 6.E.3.1: The student is able to describe models of light traveling across a boundary from one transparent
material to another when the speed of propagation changes, causing a change in the path of the light ray at the
boundary of the two media. [See Science Practices 1.1, 1.4]
LO 7.A.3.3: The student is able to analyze graphical representations of macroscopic variables for an ideal gas to
determine the relationships between these variables and to ultimately determine the ideal gas law
PV  nRT . [See Science Practices 5.1]

© 2019 The College Board.


Visit the College Board on the web: collegeboard.org.
2021

AP Physics 2:
®

Algebra-Based
Scoring Guidelines

© 2021 College Board. College Board, Advanced Placement, AP, AP Central, and the acorn logo are registered
trademarks of College Board. Visit College Board on the web: collegeboard.org.
AP Central is the official online home for the AP Program: apcentral.collegeboard.org.
AP® Physics 2: Algebra-Based 2021 Scoring Guidelines

Question 1: Short Answer 10 points


(a) For indicating that ∆U = 0J 1 point
For correctly calculating the net work done during the two processes with correct units 1 point
( ) ( )
W =− P∆V =− 100 × 103 Pa ( 4 − 1) ×103 m  =−300 J
Scoring note: The answer must either have the negative sign or indicate that the work is
done by the gas.
For substituting ∆U and W into the first law of thermodynamics to obtain a value for Q 1 point
OR for applying the first law to show that Q is equal in magnitude to W but opposite in
sign
Example response for part (a)
The change in internal energy is zero because the initial and final temperatures are the
same at points 1 and 3. The work done on the gas is -P∆V = -300 J. Because the work is
negative, 300 J of energy must be transferred to the gas by heating in order for the
internal energy of the gas to remain constant.
Total for part (a) 3 points
(b) i. For indicating that the work is less than in part (a), with a reference to less area under the 1 point
curve
For indicating that the work is positive or opposite the sign indicated in part (a), with a 1 point
reference to the sign of the change in volume or the direction of the process as indicated
in the graph
Example response for part (b)(i)
The magnitude of the work done is less than the work in part (a) because there is less
area under the curve. The work is also the opposite sign from part (a) because the volume
decreases, as shown by the direction of the arrow.
ii. For stating that there is no change in average kinetic energy/speed of gas molecules 1 point
For indicating a change that is relevant to the collision rate 1 point
Scoring note: Acceptable responses include volume, surface area, time to traverse the
container.
For indicating that there are more collisions with the walls of the container; thus, more 1 point
force per area (must refer to the walls)
Example response for part (b)(ii)
Temperature does not change, so the speed of the molecules and the force of collisions
with the walls of the container stays the same. Volume decreases, so the density of the gas
molecules increases, and they collide more frequently. This means more net force due to
collisions with the container walls. The smaller volume also means less surface area.
Total for part (b) 5 points
(c) For indicating that the temperature in state 2 is higher than in state 3 1 point
For indicating that energy flows from the state indicated as hotter to the state indicated as 1 point
cooler
Example response for part (c)
The temperature of the gas in sample 2 is higher than the temperature of sample 3.
Energy goes from hot to cold, so energy will transfer from sample 2 to sample 3.

© 2021 College Board


AP® Physics 2: Algebra-Based 2021 Scoring Guidelines
Total for part (c) 2 points

Total for question 1 10 points

© 2021 College Board


AP® Physics 2: Algebra-Based 2021 Scoring Guidelines

Question 2: Experimental Design 12 points

(a) i. For describing a valid method for keeping the temperature constant 1 point
For describing a valid use of the objects of known mass to affect the pressure 1 point
For explicitly measuring the height h and the radius r (or diameter) of the piston 1 point
For explicitly obtaining more than two data points 1 point
Example response for part (a)(i)
Place the container in an ice bath, so the part below the piston is submerged. Measure the
radius and height of the piston. For eight different objects of known mass, add each
object on the piston and measure the height of the piston for each object.
ii. For an equation that correctly relates pressure to measured quantities consistent with the 1 point
procedure in (a)(i)
Example response for part (a)(ii)
Ptot = Patm + ( m p + Nmo ) g A = Patm + ( m p + Nmo ) g (π r 2 ) , where N is the number
of objects on the piston and r is the radius of the piston.
iii. For an equation that correctly relates the density of the gas to measured quantities 1 point
Example response for part (a)(iii)
=ρ M
= g V Mg (π r 2 h )
iv. For referring to the ideal gas law or Boyle’s Law and using the equation to show pressure 1 point
and volume are inversely proportional
For a conclusion based on an analysis of the slope of the graph and a correct relationship 1 point
between pressure and density
Example response for part (a)(iv)
According to the ideal gas law, pressure is proportional to 1/V. Because the mass of this
gas is constant, pressure is, therefore, directly proportional to density. The graph does
not show a linear relationship between density and pressure, so the gas is not ideal.
Total for part (a) 8 points
(b) For indicating that the water pressure, and thus the pressure on the balloon and of the gas, 1 point
increases as the depth is increased
Example response for part (b)
When the balloon goes deeper in the fluid, the pressure increases. This will cause the
volume of the balloon to decrease.

© 2021 College Board


AP® Physics 2: Algebra-Based 2021 Scoring Guidelines

(c) For correctly applying Newton’s second law with some specific elements of the problem, 1 point
including one of the two weights
For a correct expression for the weight of the balloon and gas 1 point
For a correct expression for the buoyant force 1 point
Example response for part (c)
∑ F ==
0 FB − Wballoon − Wgas − Fstudent
FB = ρ wVb g
=W ρ gVb g + mb g
Fstudent = ρ wVb g − ( ρ gVb g + mb g )
Total for part (c) 3 points

Total for question 2 12 points

© 2021 College Board


AP® Physics 2: Algebra-Based 2021 Scoring Guidelines

Question 3: Quantitative/Qualitative Translation 12 points

(a) For indicating that the magnetic field needs to double in order to double the force (since 1 point
the force equals qvB )
For correctly explaining why the magnitude of the current must double, without any 1 point
incorrect statements
For indicating that the current must go in the opposite direction, i.e. be negative 1 point
Example response for part (a)
The current must change direction and double in magnitude. The graph shows that when
the current doubles the magnetic field doubles. When the magnetic field doubles, the
magnetic force doubles. Reversing the direction of the current will reverse the direction
of the magnetic field, and therefore the direction of the force.
Total for part (a) 3 points
(b) For indicating that the emf is the same 1 point
For indicating that larger resistance and/or less current means less power, without 1 point
making any incorrect statements
For indicating that the slope between t1 and t3 is less, or the horizontal line segment at 1 point
right would be below the one shown
Example response for part (b)
The slope of the energy vs. time graph represents power. Because the induced emf is the
same, but resistance is higher, power is lower. Therefore, the slope would be smaller.
Total for part (b) 3 points
(c) For indicating that the greater change in magnetic field means a greater change in 1 point
magnetic flux or a larger emf
For indicating that power increases with increasing emf so the power and thus the energy 1 point
dissipated is greater, or for an answer that is consistent with the response to the previous
point in (c).
For indicating that the slope between t1 and t3 is greater, or the horizontal line segment 1 point
at right would be above the one shown, or for an answer that is consistent with the
response to the first point in (c)
Example response for part (c)
The induced emf is larger than it was before because the magnetic field changed by a larger
amount in the same time period. Power is proportional to the square of the emf, so the power
is larger. Power is the slope of energy vs time, so the slope is greater for the new graph.
Total for part (c) 3 points
(d) For any indication the cumulative energy dissipated depends on the time elapsed 1 point
For indicating that the emf depends on the rate of change of the magnetic flux, i.e., it is 1 point
inversely proportional to the time elapsed
For indicating that the power is proportional to the square of the emf and the energy 1 point
dissipated is power times time, so energy is inversely proportional to the time
Total for part (d) 3 points

Total for question 3 12 points

© 2021 College Board


AP® Physics 2: Algebra-Based 2021 Scoring Guidelines

Question 4: Short Answer Paragraph Argument 10 points

(a) For obtaining the correct relationship between electron wavelength and speed 1 point
For correctly substituting values 1 point
Example response for part (a)
λ h=
= p h mv

v=
( 6.63 × 10−34 J s )
( 9.11 × 10−31 kg )( 5.0 × 10−9 m )
v 1.5 × 105 m s
=
Total for part (a) 2 points

(b) For using E = mc 2 to convert mass to energy 1 point

For including the kinetic energy of the electron 1 point


For including the equivalent energy of both particles and correct substitutions, with a 1 point
speed consistent with the answer to part (a)
Example response for part (b)
( )( ) ( )( )
2 2
2 9.11 × 10−31 kg 3 × 108 m s
Etot = + (1 2 ) 9.11 × 10−31 kg 1.5 × 105 m s

= 1.6 × 10−13 J
Etot
Total for part (b) 3 points

(c) For indicating that the photon has a component of momentum (or velocity) toward the 1 point
bottom of the page
For indicating that momentum must be conserved in both the horizontal and vertical 1 point
directions
For indicating that energy is conserved 1 point
For indicating that the new photon has less energy, so it has a lower frequency 1 point
For a logical, relevant, and internally consistent argument that addresses the required 1 point
argument or question asked and follows the guidelines described in the published
requirements for the paragraph-length response
Example response for part (c)
In order to conserve momentum in the vertical direction, the photon must have a
component of its momentum toward the bottom of the page. If the horizontal component of
the momentum of the electron after the collision is less than the initial momentum of the
photon, then the photon must move toward the right after the interaction. In order to
conserve energy, the frequency of the photon after the collision is less than what it was
before the collision because it gave some of its energy to the electron.
Total for part (c) 5 points

Total for question 4 10 points

© 2021 College Board


2022

AP Physics 2:
®

Algebra-Based
Scoring Guidelines

© 2022 College Board. College Board, Advanced Placement, AP, AP Central, and the acorn logo are registered
trademarks of College Board. Visit College Board on the web: collegeboard.org.
AP Central is the official online home for the AP Program: apcentral.collegeboard.org.
AP® Physics 2: Algebra-Based 2022 Scoring Guidelines

Question 1: Short Answer Paragraph Argument 10 points

(a)(i) For correctly drawing and labeling the gravitational, buoyant, and tension forces with no 1 point
extraneous forces
Example Response

(a)(ii) For an application of Newton’s laws that is correct or consistent with the diagram in part (a)(i) 1 point
and indicates zero net force
For a correct substitution of the buoyant force into a solution that is consistent with the previous 1 point
equation

Scoring Note: A correct answer with no supporting work earns this point.
Example Response
 
 F  ma

FT  FB  Fg  0

FT  FB  Fg

FT  Fg  FB

FT  mb g   wVb g

Total for part (a) 3 points


(b) For correctly relating the speed of the light in the new medium to the index of refraction 1 point
For indicating that the frequency does not change 1 point
For a correct relationship between speed and wavelength 1 point
For a correct relationship between wavelength and fringe separation 1 point
For a logical, relevant, and internally consistent argument that addresses the question asked and 1 point
follows the guidelines described in the published requirements for the paragraph-length response

© 2022 College Board


AP® Physics 2: Algebra-Based 2022 Scoring Guidelines
Example Response
The speed of light in the new fluid is less than the speed of light in water because the fluid has a
greater index of refraction. This means that the wavelength of the light in the beam will be
smaller because the frequency does not change. Since the wavelength is smaller, the angular
separation of the bright fringes will decrease, as described by the equ ation m  d sin  .
Total for part (b) 5 points
(c) For explicitly indicating that the amount of refraction at the fluid-prism interface depends on the 1 point
ratio of the indices of refraction of the materials
Scoring Note: Basing the explanation on the difference in refractive indices is acceptable.
For correctly relating a larger angle of refraction to the beam hitting the screen below point P 1 point
Example Response

The beam refracts more when the air is present because the difference between the indices of
refraction between the prism and the surrounding medium is greater. So, the beam hits the
screen below point P.
Total for part (c) 2 points

Total for question 1 10 points

© 2022 College Board


AP® Physics 2: Algebra-Based 2022 Scoring Guidelines

Question 2: Experimental Design 12 points

(a) For a correct ranking 1 point

1 VA 1 VB 3 VC 2 VD

For indicating that the resistors in parallel will have the same potential difference 1 point

For a justification that indicates VD  VC because RD  2RC 1 point


Total for part (a) 3 points
(b)(i) For calculating the correct value of the charge on the 200F capacitor, including units 1 point

Example Response

Q
V 
C

Q  C V   200F  0.91 V 

Q  1.82  104 C

(b)(ii) For indicating one of the following as evidence that the capacitors are in series: 1 point

 the potential differences across the capacitors are different


 the sum of the potential differences across the capacitors is constant
 the sum of the potential differences across the capacitors is approximately equal to the
potential difference across the battery

(b)(iii) For an explanation that correctly addresses one of the following: 1 point

 that the potential differences across the known and unknown capacitor will always be
the same
 that the charge on the unknown capacitor cannot be determined

Example Response

Both charge and potential difference across the capacitor are needed to determine C.
Arranging the capacitors in parallel will mean both capacitors will have the same potential
difference. However, capacitors in parallel will have differing amounts of charge, making it
impossible to determine the charge, and, therefore, the capacitance of the unknown
capacitor.

Total for part (b) 3 points

© 2022 College Board


AP® Physics 2: Algebra-Based 2022 Scoring Guidelines
(c)(i) For choosing two quantities that will produce a linear plot that can be used to find CU 1 point

Example Responses

 Qknown (Cknown Vknown ) and VU


 QU and VU
VU
 Cknown and
Vknown

(c)(ii) For labeling the axes and including appropriate units consistent with Part (c)(i) 1 point
For correctly plotting points (with valid scaling consistent with units) so that the plotted 1 point
points cover at least half of the grid’s width and height
For drawing an appropriate linear best-fit line 1 point
Example Response

© 2022 College Board


AP® Physics 2: Algebra-Based 2022 Scoring Guidelines
(c)(iii) For using points on the best-fit line to calculate the slope of the line 1 point
For correctly determining the capacitance from the slope of the line 1 point
Example Response

Capacitance is equal to slope

212 μC  190 μC
CU 
4.10 V  3.67 V
CU  51.2 μF

Total for part (c) 6 points

Total for question 2 12 points

© 2022 College Board


AP® Physics 2: Algebra-Based 2022 Scoring Guidelines

Question 3: Quantitative/Qualitative Translation 12 points

(a) For indicating that the electrostatic force is equal to the net (centripetal) force on the electron, 1 point
with a correct expression for each

Example Response
 
 F  ma
FE  FC
kq 2 mv 2

r2 r

Scoring Note: An incorrect mass label is acceptable to earn this point.

For using the expressions for the electrostatic and net forces to determine the speed v of the 1 point
electron (responses must indicate that the mass in the expression represents the mass of the
electron and the charge in the expression represents the charge of the electron)

Example Response

ke2 me v 2

r2 r
ke 2
v2 
me r

ke 2
v
me r

Scoring Note: qe and q p are acceptable.

Total for part (a) 2 points


(b) For a correct expression for electric potential energy, using charges consistent with charges 1 point
from part (a)

Example Response

ke 2
U 
r
For a correct expression for kinetic energy of the electron, including a substitution consistent 1 point
with the expression from part (a) to eliminate speed from the equation

Example Response

1  ke 2  1 ke 2
K me  
2  me r  2 r

© 2022 College Board


AP® Physics 2: Algebra-Based 2022 Scoring Guidelines
For indicating that the total energy of the atom is the sum of the electric potential energy and 1 point
the kinetic energy of the electron

Example Response

E U  K
ke2 1 ke 2
E 
r 2 r
2
ke
E
2r
Total for part (b) 3 points
(c) For correctly indicating consistency between the equation in part (b) and the description with 1 point
an explanation that references the equation in part (b)
For correctly addressing functional dependence of the energy equation from part (b) to the 1 point
orbital radius of the electron
Example Response

The equation from part (b) indicates that as the radius increases, the total energy of the atom
becomes less negative, which is an increase in the total energy. This is consistent with the
given description of the atom absorbing a photon.
Total for part (c) 2 points
(d)(i) For a correct calculation of the energy of the photon 1 point

Example Response
E  hf
E  (6.63  1034 J  s)(3.2  1015 Hz)
E  2.12  1018 J
(d)(ii) For a correct calculation of the mass-energy of an electron 1 point

Example Response

E  mc 2
E  (9.11  1031 kg)(3.00  108 m/s) 2
E  8.20  1014 J
For correctly indicating that the photon energy is negligible compared to the mass energy of 1 point
the electron (or an answer that is consistent with the energies calculated in part (d)(i) and
part (d)(ii))

Scoring Note: The energy comparison must be from the unit of joules to joules or the unit of
electron volts to electron volts in order for this point to be earned.

© 2022 College Board


AP® Physics 2: Algebra-Based 2022 Scoring Guidelines
(d)(iii) For U smaller in magnitude but still negative 1 point
For K smaller in magnitude but still positive 1 point
Example Response

Total for part (d) 5 points

Total for question 3 12 points

© 2022 College Board


AP® Physics 2: Algebra-Based 2022 Scoring Guidelines

Question 4: Short Answer 10 points

(a) For an appropriate use of Newton’s laws to set the magnetic force equal to the electric force 1 point
For using correct expressions for the magnetic and electric forces 1 point
For substituting an expression for the magnetic field to yield a correct expression that 1 point
includes v and the given quantities
Example Response
 
F  ma

FM  FE  0

FM  FE

qvB  qE

 I
v  0   E
 2 d 

2 dE
v
0 I

Total for part (a) 3 points


(b)(i) For an “X” between point P1 and the dashed line 1 point

Example Response

For indicating that the magnetic field strength is inversely proportional to the distance from 1 point
the wire
Example Response

Magnetic field is inversely proportional to the distance from a long, straight current carrying
 I
wire: B  0 . Doubling the distance from the wire from L to 2L would reduce the
2 r
magnetic field from 3B0 to 1.5B0 . Therefore, the magnetic field would be equal to 2B0
somewhere between L and 2L .

© 2022 College Board


AP® Physics 2: Algebra-Based 2022 Scoring Guidelines
(b)(ii) For using the change in flux, with correct substitutions, to determine the emf 1 point
For correctly applying Ohm’s law with correct substitutions 1 point
Scoring Note: It is not necessary to independently calculate a numerical value for the emf.

Example Response

  
 B

 
5.0  105  1.0  105 T m 2
 2.0  105 V
t 2.0 s

I 
 
2.0  105 V
 2.0  106 A
R 10 

Total for part (b) 4 points


(c) For indicating that the current in the round coil produces a magnetic field 1 point

For indicating that the magnetic field from the round coil produces a flux through the square 1 point
coil

For indicating that the changing flux produces an emf or current in the square coil circuit 1 point

Scoring Note: A response that indicates that the magnetic flux only changes during a portion
of the entire time interval does not earn this point.
Example Response

The current in the round coil produces a magnetic field. The magnetic field from the round
coil passes through the square coil, producing a flux. As the current in the power supply
increases, so does the current in the round coil, and, therefore, the magnetic field created by
the current increases. Since the magnetic field changes, the flux through the square coil
changes. The constantly changing magnetic flux through the square coil produces an emf
and, therefore, current in the square coil to light the lightbulb.
Total for part (c) 3 points

Total for question 4 10 points

© 2022 College Board

You might also like